Explaining this was was one of the key initial achievements of the Dirac equation.
Yes, but this is not predicted by the Schrödinger equation, you need to go to the Dirac equation.
See also:
- physics.stackexchange.com/questions/233330/why-do-electrons-jump-between-orbitals
- physics.stackexchange.com/questions/117417/quantum-mechanics-scattering-theory/522220#522220
- physics.stackexchange.com/questions/430268/stimulated-emission-how-can-giving-energy-to-electrons-make-them-decay-to-a-low/430288
New to topics? Read the docs here!